THE SAT WRITING AND LANGUAGE TEST: THE TEN ESSENTIAL RULES - McGraw-Hill Education SAT 2017 Edition (Mcgraw Hill's Sat) (2016)

McGraw-Hill Education SAT 2017 Edition (Mcgraw Hill's Sat) (2016)

CHAPTER 4

THE SAT WRITING AND LANGUAGE TEST: THE TEN ESSENTIAL RULES

  1. Don”t Sweat the Small Stuff
  2. Strengthen the Core
  3. Organize the Ideas in Your Paragraphs
  4. Use Parallel Structure
  5. Use Modifiers Effectively
  6. Make Your Comparisons Clear and Precise
  7. Make Sure Your Pronouns Are Clear and Precise
  8. Make Your Verbs Clear and Precise
  9. Make the Rest of Your Sentence Clear and Precise
  10. Know How to Punctuate

The SAT Writing and Language Test

What is the SAT Writing and Language test?

The SAT includes a 35-minute Writing and Language test designed to assess your

proficiency in revising and editing a range of texts in a variety of content areas, both academic and career related, for expression of ideas and for conformity to the conventions of Standard Written English grammar, usage and punctuation .

The Writing and Language test consists of four passages, each 400–450 words long, in the categories of careers, social studies, humanities, and science. (For an example of the Writing and Language test, look at Section 2 of the Diagnostic Test in Chapter 2 .) You are to analyze underlined portions of each passage and to determine whether they need to be revised according to the standards of

  • parallel structure
  • verb, modifier, and pronoun agreement
  • standard idiom
  • logical comparisons
  • word choice
  • verb tense, mood, and voice
  • logical transitions
  • coordination of ideas
  • punctuation

You are also asked more general editorial questions, such as

  • whether a certain sentence adds to or detracts from the cohesiveness of a paragraph
  • where a new sentence should be placed for maximum effectiveness
  • whether a particular passage or paragraph has the effect the author intends

How is it used?

Colleges use your SAT Writing and Language test score as a measure of your ability to write clearly and effectively. Good writing skills are essential to success in the liberal arts and sciences. The Writing and Language test score represents one-half of your Evidence-Based Reading and Writing Score. The other half of this score comes from the Reading test.

Sound intimidating? It”s not.

There are really only 10 rules to learn in order to ace the SAT Writing and Language test, and the 33 lessons in this chapter will give you the knowledge and practice you need to master all of them.

Rule 1: Don”t Sweat the Small Stuff

Lesson 1: Know the seven things to NOT worry about

  1. Don”t worry about split infinitives

Which is correct?

  1. Here are seven s tonot worry about .
  2. Here are seven thingsnot to worry about .

Sentence A includes a split infinitive : the infinitive to worry has an adverb (not ) wedged inside it. Although the SAT probably won”t test your skill for “unsplitting” infinitives, you should still do it as a matter of politeness to the grammar scolds, for whom they are the verbal equivalent of chewing aluminum foil. You can usually just shift the adverb over a little bit, as in sentence B, and make everyone happy.

But sometimes it”s not so easy to unsplit infinitives without destroying the tone or meaning of the sentence. For instance, try unsplitting the infinitive in The company plans to more than double its revenue next year . Or, better yet, just don”t worry about it, since it won”t be on the SAT.

  1. Don”t worry (too much) aboutwho vs. whom

Which is correct?

  1. Towho should I give your condolences?
  2. Towhom should I give your condolences?

The who/whom distinction is the same as the he /him and they /them distinction: the first pronoun in each pair has the subjective case (Lesson 21), and so is used as the subject of a verb, and the second has the objective case , and so is used as the object of a verb or preposition. Since the pronoun in the sentence above is the object of the preposition to , sentence B is correct.

Notice, however, that the pronoun you can be used as either a subject or an object. It represents a “merger” between the subjective thou and the objective thee from Elizabethan English. (Remember Shakespeare?) Likewise, whomseems to be in the process of merging with who . For instance, even Standard English allows a sentence like Who are you talking to? rather than insisting on the rather uptight-sounding To whom are you talking?

The bottom line? Chances are, your SAT won”t ask you to choose between who and whom . But if it does, just remember that the who/whom distinction is the same as the they/them and he/him distinctions. And if you”re still stuck, just go with who .

  1. Don”t worry aboutthat vs. which

Which is correct?

  1. Second Federal is the only bank in townwhich does not finance commercial mortgages .
  2. Second Federal is the only bank in townthat does not finance commercial mortgages .

Technically, sentence B is correct because the phrase that does not finance commercial mortgages is a “restrictive clause,” that is, it modifies the noun bank by attaching a defining characteristic to it. If a modifying clause is “restrictive” (that is, it conveys defining information about the noun), it should use that . Alternately, if the clause is “nonrestrictive” (that is, it conveys incidental or nondefining information about the noun), it should use which . Helpful tip: nonrestrictive modifying clauses are almost always preceded by a comma, as in The speech, which lasted only three minutes, secured her reputation as a master orator .

Bottom line: the SAT will probably not expect you to distinguish restrictive from nonrestrictive clauses, so don”t stress out about that versus which on the SAT.

  1. Don”t worry about starting sentences withBecause, And , or But

Which is correct?

  1. Because we don”t know when Jennie will arrive, we can”t make dinner reservations yet.
  2. We can”t make dinner reservations yet because we don”t know when Jennie will arrive.

Ms. Bumthistle (everyone”s fifth grade English teacher) probably told you that it”s a cardinal sin to start a sentence with Because , And , or But . But it”s not nice to lie to children. In fact, either sentence above is fine. The SAT frequently includes perfectly good sentences that start with Because . But if you want to avoid annoying the Ms. Bumthistles of the world, avoid the practice in your own writing if it”s not too much trouble.

  1. Don”t worry about disappearingthat s

Which is correct?

  1. I really love the sweater you gave me.
  2. I really love the sweaterthat you gave me .

Both of the sentences above are acceptable in Standard Written English. So, if that isn”t necessary, why would we ever include it? Because it takes some of the burden away from sweater , which is an object in the first clause (I really love the sweater ) as well as an object of the second clause (You gave me [the sweater] ). By including that , we separate the two ideas more clearly. But since very few people are confused by the dual role of sweater in the first sentence, that is not strictly necessary.

Bottom line: don”t worry about a missing that , as long as the resulting sentence still makes sense.

  1. Don”t worry about “parallel ellipsis”

Which is correct?

  1. The Republicans reacted to the speech with sustained applause; the Democrats, however, reacted to it with studied silence.
  2. The Republicans reacted to the speech with sustained applause; the Democrats, studied silence.

Both of the sentences above are grammatically correct. Sentence B, however, is more concise because it takes advantage of “parallel ellipsis.” Ellipsis simply means the omission of words that are implied by context. In this case, the parallel structure of the two clauses allows the reader to “fill in” the missing words.

When you read a sentence like B, you might think that the missing words are a grammatical mistake. But if the context clearly implies the missing words, you can leave them out.

You might notice that, in sentence B, the comma plays an unusual role. Usually, commas are used to separate items in a list, to separate modifying phrases from clauses, or (with conjunctions) to separate clauses. Here, however, the comma is analogous to the apostrophe in can”t : just as the apostrophe holds the place of the missing letters from cannot , so the comma in sentence B holds the place of the missing words (however, reacted to it with ) from sentence A. Without that comma to suggest the ellipsis, the sentence would sound very strange indeed.

  1. Don”t worry (too much) aboutgood versus well or bad versus badly

Which is correct?

  1. Peter performedgood .
  2. Peter performedwell .

Here, performed is an action verb. Any word that modifies the manner of an action verb is an adverb . Since good cannot function as an adverb in Standard English, only choice B is correct.

Which is correct?

  1. I don”t feelgood .
  2. I don”t feelwell .

Here, feel is a linking verb rather than an action verb: that is, it links the subject to an essential adjective, as in The sky is blue . So does this mean that C is right and D is wrong? No—they are both grammatically and semantically correct, since well can also act as an adjective, meaning “in good health.” The two sentences are essentially equivalent to I am not [feeling] good and I am not well .

Which is correct?

  1. I feelbad for you .
  2. I feelbadly for you .

Here, despite what your know-it-all friends might say, E is correct and F is wrong, since badly can only function as an adverb. Saying I feel bad for you is like saying I feel sorry for you . You wouldn”t say I feel sorrily for you , would you?

It”s important to know the difference between adjectives and adverbs (Lesson 14), and between action verbs and linking verbs.

But the SAT is probably not going to ask you about good versus well or bad versus badly .

Rule 2: Strengthen the Core

Lesson 2: Identify your clauses, modifiers, and conjunctions

The first and most important step in analyzing sentences is identifying clauses.

Every sentence contains at least one clause , which consists of a subject and a predicate . The subject is the noun or pronoun that is “doing” the verb, and the predicate consists of a verb and its complements (such as direct objects, indirect objects, verb modifiers, or predicate adjectives).

The subject-verb unit of any clause conveys the core idea of that clause. For instance, if we take the sentence

As the sun slowly set, the desperation of the sailors revealed itself in their sullen glances .

and isolate just the subject and verb, we still retain the core idea:

The desperation revealed itself .

Consider these two sentences:

  1. Go!
  2. Although generally regarded as the most daunting course in the undergraduate science curriculum, Introduction to Organic Chemistry not only provides a necessary foundation in the principles of physical chemistry, but also introduces students to important experimental methods at the heart of today”s most promising areas of medical research.

Sentence A is the shortest in the English language. It has everything necessary to convey a complete thought: a verb (go ) and its subject (the implied subject you ). Since it is in the imperative mood (Lesson 30), the subject is assumed to be the person being addressed and does not need to be stated.

So here”s how we can analyze sentence A:

[You] [go] !

[Implied subject ] [verb ]!

Sentences can also elaborate the main clause with modifiers or link clauses with conjunctions.

Sentence B is a bit more complicated. The main clause includes a compound predicate, so it combines two statements with the same subject into one sentence:

Introduction to Organic Chemistryprovides a necessary foundation in the principles of physical chemistry

Introduction to Organic Chemistryintroduces students to important experimental methods at the heart today”s most promising areas of medical research .

These two clauses are linked by a conjunction phrase (not only … but also ), and are preceded by a subordinating conjunction (Although ) followed by a modifying (participial) phrase (generally regarded as the most daunting course in the undergraduate science curriculum ). We”ll talk more about conjunctions in Lessons 6 and 10 and about participial phrases in Lesson 12.

So here”s how you should analyze sentence B:

[Although ] [generally regarded as the most daunting course in the undergraduate science curriculum ], [ Introduction to Organic Chemistry ] [not only ] [ provides a necessary foundation in the principles of physical chemistry ], [but also ] [ introduces students to important experimental methods at the heart today”s most promising areas of medical research .]

[Subordinating conjunction] [participial phrase], [subject ], [conjunction part 1] [predicate 1 ] [conjunction part 2] [predicate 2 ].

If this analysis seems confusing now, don”t worry. We”ll explain all of these terms in the lessons to come. For now, focus on identifying clauses (the words in bold in the sentence above) because they are the core of any sentence. Distinguishing clauses from the rest of the sentence is the first step to becoming a stronger reader and writer.

Lesson 3: Trim every sentence to analyze its core

Consider this sentence:

My chief concern with this budget, which has otherwise been well considered, are the drastic cuts in school funds .

How does it sound? It may sound a little bit off, but why, and how do we improve it? This is where trimming comes in.

Diagnosing and improving sentences requires mastering the skill of trimming : reducing the sentence to its core , or its essential elements, then analyzing that core.

This is based on a very important rule of grammar: every sentence must “work” even when its prepositional phrases, interrupters, and other modifiers are eliminated. That is, it still must convey a grammatically complete idea.

Step 1: Cross out all nonessential prepositional phrases.

A preposition is any word that can be used to complete any sentence like these:

The squirrel ran _____the tree . (e.g. up , to , around , from , in , by , on , into , etc.)

I went to the party _____ a brain surgeon . (e.g., as , with , for , etc.)

Democracy is government _____ the people . (e.g., for , of , by , etc.)

A prepositional phrase is the preposition plus the noun phrase that follows it, such as from sea to shining sea, in the beginning , and for the money .

Our sentence has two nonessential prepositional phrases that we can eliminate:

My chief concern with this budget , which has otherwise been well considered, are the drastic cuts in school funds .

Step 2: Cross out all interrupting modifiers.

Interrupting modifiers are generally easy to spot because they come between commas or dashes. The sentence should always hold together even when the interrupting modifiers are removed:

My chief concern with this budget, which has otherwise been well considered , are the drastic cuts in school funds .

Step 3: Cross out any other nonessential modifiers.

Once you learn to identify participial phrases (Lesson 12), appositives (Lesson 13), and more mundane modifiers like adjectives and adverbs (Lesson 14), you can trim them from all of your sentences, as well, with one exception: predicate adjectives , such as tired in the sentence Karen was tired , without which the sentence doesn”t convey an idea. In our sentence, chief and drastic can go:

My chief concern with this budget, which has otherwise been well considered , are the drastic cuts in school funds .

So now we have the core:

My concern are the cuts .

Obviously, the subject and verb disagree (Lesson 4): concern is a singular subject, but are is a plural verb. So you may just want to change the verb: My concern is the cuts . But that”s no good either, because now the sentence has a number shift (Lesson 11): the singular concern is equated with the plural cuts .

These problems point to an even deeper problem: the most essential part of the sentence, the verb, is very weak. Forms of the verb to be , like is , are , was , and were , are among the weakest verbs in English.

To improve your writing, first focus on strengthening and clarifying your verbs .

This sentence is clearly indicating disapproval, so a more personal subject like I and a strong verb of disapproval like object would strengthen the sentence:

Although the budget is otherwise well considered, I object to its drastic cuts in school funds .

Notice that this revision not only corrects the grammatical problems, but it also makes the sentence stronger, clearer, and more concise.

Exercise 1: Trimming and Strengthening Sentences

Trim each of the following sentences and correct any verb problems.

1 . The team of advisors, arriving slightly ahead of schedule, were met at the airport by the Assistant Prime Minister.

2 . The flock of birds darting over the roiling lake look like an opalescent whirlwind.

3 . Carmen, not to mention her unsympathetic sisters, were unaffected by David”s pleas.

4 . Juggling the demands of school, family, and work often seem too much for a young mother to bear.

5 . Others on the committee, like chairman Sanders, is concerned about the lack of attention given to school safety.

6 . Every one of my friends, including the boys, has supported my decision.

7 . The fact that human institutions have been responsible for so many atrocities have forced some historians to adopt a cynical perspective on human nature.

Trim each sentence. Then revise it to make it clear and concise, changing the subject and verb, if necessary.

8 . The progression of a society, or at least that popularly regarded as advancements, are a result of gradual modifications, not sudden drastic overhaul.

Trimmed: _________________________

Revised: __________________________

__________________________________

9 . The development of the new country”s government and social institutions were affected in a negative regard by the lack of cohesiveness within the revolutionary army.

Trimmed: _________________________

Revised: __________________________

__________________________________

10 . This report is intended for presenting arguments in opposition to what I took to be the less than optimal response of the administration to the most recent crises in the Middle East.

Trimmed: _________________________

Revised: __________________________

__________________________________

Lesson 4: Make sure your verbs agree with their subjects

Which is correct?

  1. Data gathered through pollingis not as reliable as data gathered objectively .
  2. Data gathered through pollingare not as reliable as data gathered objectively .

If we trim sentence A, we get

Data gathered through polling is not as reliable as data gathered objectively .

The subject, data , is plural, so the verb should be are . Sentence B is correct.

A few Latin plurals are frequently mistaken for singulars. Don”t make that mistake.

Which is correct?

  1. Behind every successful work of artlies countless hours of toil and trial .
  2. Behind every successful work of artlie countless hours of toil and trial .

If we trim sentence C, we get

Behind every successful work of art lies countless hours of toil and trial .

Here, the subject and verb are inverted : the subject hours comes after the verb lies . When we “un-invert” the clause, the subject-verb disagreement is obvious: hours lies should be changed to hours lie . Therefore, sentence D is correct.

An inverted clause , where the verb comes before the subject, usually begins with the dummy subject there , as in There is or There are , or is preceded by a prepositional phrase.

Every inverted clause can be “un-inverted” by removing any dummy subject and rearranging the phrases. Un-inverting these sentences will help you to spot any subject-verb disagreements.

Which is correct?

  1. One or two of my classmateshas a strong chance of winning an award .
  2. One or two of my classmateshave a strong chance of winning an award .

If we trim sentence E, we get

One or two of my classmates has a strong chance of winning an award .

Is the subject, One or two , singular or plural? In these ambiguous situations, it helps to remember the law of proximity : the essential noun (that is, not one in a prepositional phrase) that is closer to the verb gets priority. Here, since two is closer to the verb, the subject is regarded as plural. Therefore, sentence F is correct.

If a subject takes the form a or b , it is assumed to take the number of b .

Exercise 2: Subject-Verb Agreement

Choose the correct verb form.

1 . The flock of geese (was/were) startled by the shotgun blast.

2 . The data on my computer (was/were) lost when the hard drive failed.

3 . Neither of the twins (is/are) allergic to penicillin.

4 . Much of what I hear in those lectures (go/goes) in one ear and out the other.

5 . Amy, like her friends Jamie and Jen, (wants/want) to go to Mount Holyoke College.

6 . Among the lilies and wildflowers (were/was) one solitary rose.

7 . Either the chairperson or her assistants (is/are) going to have to make the decision.

8 . There (is/are) hardly even a speck of dirt left on the carpet.

9 . In every teaspoon of soil (are/is) over two million tiny microorganisms.

10 . There (is/are), in my opinion, far too few primary physicians working in this district.

11 . Beyond that hill (is/are) hundreds of bison.

12 . Never before (have/has) there been such voices heard on the public airwaves.

13 . Every player on both teams (was/were) at the press conference after the game.

14 . There (has/have) been a theater and a toy store in the mall ever since it opened.

15 . There (is/are) a great many production problems to iron out before show time.

16 . The proceeds from the sale of every auctioned item (goes/go) to charity.

17 . There (is/are) more than three years remaining on her contract.

18 . Neither of the girls (was/were) frightened by the small animals that scurried past their tent.

19 . This technology, developed by the military for field communications, (have/has) become essential to private industry as well.

20 . Every player on both teams (was/were) concerned about the goalie”s injury.

21 . The company”s sponsorship of mentorship programs (has/have) garnered many accolades from other philanthropic organizations.

22 . Neither the children nor their parents (utter/utters) a word when Mrs. Denny tells her stories.

23 . How important (is/are) strength training and cardiovascular training to your daily fitness regimen?

Rule 3: Organize the Ideas in Your Paragraphs

Lesson 5: Present your ideas cohesively and with a consistent tone

What”s wrong with this paragraph?

The politics of hydraulic fracturing, or “fracking,” have obscured both the dangers and the benefits of this new technology. Opponents suggest that the high-pressure fluid used to fracture deep rock formations may contain carcinogens that may seep into groundwater, and that fracking induces earthquakes. Supporters point out that this activity is taking place well below even the deepest aquifers and is well sealed off from human water supplies. The technical term for earthquakes is seismic activity, and the fractures are pretty small, really: only about 1 millimeter or less .

The paragraph starts off well, with a clear topic sentence about the politics of fracking. It then gives a quick summary of the two positions on the topic. With the last sentence, however, the paragraph begins to lose its focus and tone: the phrase pretty small, really is too conversational for the tone of this paragraph, and the ideas in the last sentence are not tied logically to the ongoing discussion. Here”s a revision that more effectively links to the previous sentence:

They also point out that the seismic activity induced by fracking is minimal: the vast majority of the fractures it induces are less than 1 millimeter wide .

Every effective prose paragraph should

  • be focused on a topic sentence that develops the central idea of the passage
  • explain or illustrate any significant claims
  • avoid irrelevant commentary
  • maintain a consistent and appropriate tone

Lesson 6: Coordinate your clauses effectively and avoid commas splices

Which is better?

  1. Despite being a best-selling author, Brian Greene is a professor of physics, he is also cofounder of the World Science Festival, and this event draws nearly half a million people each year.
  2. Cofounded by best-selling author and professor of physics Brian Greene, the World Science Festival draws nearly half a million people each year.

It”s not too hard to see that sentence B seems clearer and more logical than sentence A, but why? The answer is coordination . Both sentences contain the same four ideas, but sentence B coordinates those ideas more effectively. Sentence A contains three independent clauses:

Brian Greene is a professor of physics

… [Brian Greene] is also cofounder of the World Science Festival

[the World Science Festival] draws nearly half a million people each year

So the reader is left confused: what is the central idea of this sentence? Brian Greene”s professorship? His festival? The popularity of the festival? Even worse, the preposition Despite doesn”t make sense, since being a best-selling author doesn”t interfere in any obvious way with being a physics professor.

Sentence B, in contrast, packages these ideas to make them easier to digest. The first two ideas are subordinated in a participial phrase , and the third idea is emphasized as the independent clause .

In a well-coordinated sentence,

  • the central idea is expressed in the mainindependent clause
  • secondary ideas are expressed insubordinate clauses or modifying phrases
  • ideas are linked with logically appropriateconjunctions , prepositions , and adverbs

Notice also that the second comma in sentence A is a comma splice , joining two independent clauses. That”s a no-no.

Avoid comma splices . A comma splice is the error of joining two independent clauses with only a comma:

Independent clauses can be joined in one sentence in one of three acceptable ways:

Semicolons are used to join two ideas when the second supports or extends the first. Colons are used to join two ideas when the second explains or specifies the first. The first sentence indicates that T.J.”s guitar didn”t helpthe mood; the second indicates that T.J.”s guitar didn”t hurt the mood; the third indicates that T.J.”s guitar explained the mood.

Exercise 3: Coordinating Clauses

Join each set of sentences into a single well-coordinated sentence.

1 . H. K. Schaffer”s latest movie has received widespread critical acclaim. She directed the movie. It is the third movie that she has directed. She is the daughter of famous screenwriter George Schaffer. Her latest movie is a comedy entitled The Return .

2 . Scientists have made an important discovery. The scientists who made the discovery are a team from universities and research institutions from all over the world. The discovery concerns a region of the brain called the prefrontal cortex. The scientists have discovered that this region governs impulse control in humans. Studying this region of the brain can help scientists learn more about criminal behavior.

Rewrite the following sentences so that the clauses coordinate logically and concisely.

3 . Electric cars may not be as environmentally friendly as they are widely regarded, so the electricity they use actually comes from fossil fuels; that electricity is produced in power plants that often burn coal or other fossil fuels and that burning often produces enormous amounts of greenhouse emissions.

4 . Although regular exercise is good for your muscles, it is also good for your heart, so it is good for your brain too by keeping it well oxygenated and the increased oxygenation helps it work more efficiently.

5 . We are motivated by our principles; our principles change all the time, though: our experiences and our priorities evolve as we grow.

Lesson 7: Give your reader helpful transitions, especially between paragraphs

Consider this transition between paragraphs:

… and so we should be respectful of other people, even those with whom we disagree, while always striving to eliminate inequities and abuses of power .

To Kill a Mockingbird was written by Harper Lee and published in 1960. It portrays the fictional town of Maycomb, Alabama …

The end of the first paragraph makes a bold claim: that we should strive to eliminate inequities and abuses of power . But the next paragraph abruptly shifts to mundane facts about the publication of a particular book. Although readers who are familiar with To Kill a Mockingbird might have an idea why this author is mentioning it, the author does not provide any helpful transitions to guide the reader into the new paragraph and indicate how the new paragraph connects with previous one. Consider this revision:

… and so we should be respectful of other people, even those with whom we disagree, while always striving to eliminate inequities and abuses of power .

In To Kill a Mockingbird (1960), Harper Lee depicts a fictional town, Maycomb, Alabama, that is tainted by such inequities and abuses …

Now we understand the reference better because the author has provided a helpful paragraph transition . The phrase such inequities and abuses demonstrates clearly that the events in To Kill a Mockingbird will illustrate the importance of fighting inequities and abuse, and therefore exemplify the thesis from the previous paragraph.

Provide your readers with helpful paragraph transitions to clarify the links between topic ideas. Keep in mind the common transitional words and phrases below.

Lesson 8: Make your cross-references clear

Consider these sentences from our “fracking” essay:

… The opponents of fracking are correct to ask questions about the safety and sustainability of this process. Could it poison the local water supply with carcinogens? Can we spare the vast amount of injection water it requires? Can we safely recycle its waste-water? Could it be introducing more methane into the water supply than would naturally be present? Could it be causing potentially dangerous seismic activity? But this also must be followed by careful, scientific, and impartial investigation, not mere fear-mongering .

Each of the five questioning sentences contains the pronoun it , which makes a “cross-reference” to a noun in the first sentence, namely, fracking (or, equivalently, process ). The last sentence also includes a cross-referencing pronoun, this . But to what does it refer? It doesn”t seem to be referring to fracking anymore; that wouldn”t make sense. Nor does it make sense to refer to the other singular nouns in previous sentences, like methane , water supply , or seismic activity . So the reader may be left a bit confused. Here, we need to revise to clarify the cross-reference:

But this questioning must be followed by careful, scientific, and impartial investigation, not mere fear-mongering .

When referring to concepts introduced in previous sentences, using pronouns will often help you be concise, but make sure your cross-references are clear . Sometimes clarity may require you to replace the “cross-referencing” pronouns with more precise nouns.

Exercise 4: Effective Transitions and Cross-References

Rewrite the second sentence in each pair, providing an effective transition and clarifying any cross-references.

1 . … Modern biologists have tried for decades to explore the relationship between ancient humans and Neanderthals, but analyzing DNA from prehistoric hominids has until recently proven very difficult.

The “clean room” at the Max Planck Institute in Germany is like those used in the manufacturing of computer chips or space telescopes, solving the issue by preventing contamination from dust particles so that biologists can extract and examine minute bits of DNA from 400,000-year-old hominid bones.

2 . … As satisfying as it may be to punish wrongdoers, the real impetus behind tough sentencing laws is the belief that they actually deter crime.

The treatment so many prisoners receive in state and federal penitentiaries, including humiliation and loss of autonomy, only exacerbates any short- or long-term psychological issues that make them susceptible to antisocial and criminal impulses, according to evidence.

3 . … It”s easy to understand, in a society as complex, diverse, and bureaucratic as ours, how some citizens could develop a deep distrust of governmental institutions.

The willingness to equate all governmental institutions with tyranny is an enormously dangerous one that can only impede human moral, economic, and cultural progress.

Rule 4: Use Parallel Structure

Lesson 9: Understand the Law of Parallelism

Which is better?

  1. In the ”70s and ”80s, high school math teachers taught almost exclusively by lecture; today, more cooperative and project-based methods are likely to be employed.
  2. In the ”70s and ”80s, high school math teachers taught almost exclusively by lecture; today, they are more likely to use cooperative and project-based methods.

Which is better?

  1. Ms. Kelly always tried to provide clear instructions that showed respect and were fair to all of her students.
  2. Ms. Kelly always tried to provide instructions that were clear, respectful, and fair to all of her students.

Sentences A and C don”t seem glaringly wrong, but B and D sound a bit better. Why? Parallelism.

The Law of Parallelism

When a sentence includes a list, contrast, or comparison, the items being listed, contrasted, or compared should have the same grammatical form .

Sentence A contains two clauses that contrast teaching in the ”70s and ”80s with teaching today. However, the comparison is not parallel: the first sentence is in the active voice , but the second is in the passive voice (Lesson 29). Sentence B reads more smoothly because both clauses are in the active voice, which aligns the subjects and clarifies the contrast.

Sentence C ascribes three adjectives to Ms. Kelly”s instructions, but not in a parallel form. Sentence D clarifies the central idea by putting these adjectives in a clear and parallel list.

Lesson 10: Watch for standard parallel constructions

Which is better?

  1. It seems sometimes that our representatives would rather generate sound bites for their partisans instead of working to solve our social and economic problems.
  2. It seems sometimes that our representatives would rather generate sound bites for their partisans than solve our social and economic problems.

The problem in sentence A is hard for most readers to catch. It may take a few readings before you notice it. The word rather indicates that the sentence is making a contrast. Such a contrast requires a standard parallel construction : rather X than Y . When you see the word rather , you should expect the word than to appear soon afterward. But in sentence A, not only does than not appear, but the two words from X and Y that should be parallel are not: generate is a present-tense verb, but working is a gerund. Sentence B makes the correction, and creates the parallel construction rather generate … than solve .

Use the following standard parallel constructions precisely. When you use any of these phrases, use the precise wording, and make sure X and Y are parallel.

Lesson 11: Avoid number shifts

If a sentence equates two things, those things should have the same number .

Which is better?

  1. Everyone enjoyed their meal.
  2. Everyone enjoyed his or her meal.
  3. They all enjoyed their meals.

Sentence A commits a number shift : the pronoun their is plural, but its antecedent everyone is singular. Additionally, the object meal is singular, which doesn”t make sense—are multiple people sharing a single meal? One way to correct this problem is by changing their to the singular his or her , as in sentence B. But this phrase is needlessly awkward. Sentence C avoids both problems, so it is the best of the three.

Consider this sentence:

The problem with this plan is all of the permits we would have to file before starting the project .

If we trim it a bit, we get

The problem with this plan is all of the permits we would have to file before starting the project .

Again, we have a number shift: the singular problem is equated with the plural all of the permits . We could try to fix the problem by pluralizing the subject:

The problems with this plan are all of the permits we would have to file before starting the project .

But that sounds very strange. The best revision strengthens the verb to avoid the number shift:

Filing all of the permits required by this plan will probably delay the project .

Exercise 5: Parallel Structure

Rewrite each sentence to improve its parallel structure.

1 . The candidate”s platform included tax code reform, an improved school system, and reviving good relations with the unions.

2 . Good study practices are not so much about working hard, but rather how well you use your time.

3 . The more you get to know her, the more likely it is that you will like her.

4 . The food here is not only exceptionally fresh, but its price is also very reasonable.

5 . The financial crisis of 2007 was exacerbated by the esoteric nature of certain financial instruments, skittish investors, and the lack of awareness of regulators.

6 . Ms. Bennett is appreciated by her colleagues because she is very supportive and has a lot of knowledge.

7 . I can”t decide whether I should give Maria the tickets, or Caitlyn.

8 . The United States experienced a contraction in wealth, an increase in risk spreads, and the credit markets were deteriorating.

9 . I prefer the romantic virtuosity of Liszt, as opposed to Chopin”s emotional accessibility.

10 . The festival draws crowds from across the country that come not so much for the music but rather because of the spirit of free expression.

Rule 5: Use Modifiers Effectively

Lesson 12: Don”t let your participles dangle

Which is correct?

  1. Widely considered one of the most challenging pieces for piano, Franz Liszt stretched the boundaries of musical technique with hisEtude no. 5.
  2. Widely considered one the most challenging pieces for piano, Franz Liszt”sEtude no. 5 stretches the boundaries of musical technique .

Sentence A includes a dangling participle . The past participle considered requires a subject. Since participial phrases don”t include their own subjects, they must “borrow” them from the main clause. What is the subject of the participle? That is, what, exactly, is considered one of the most challenging pieces for piano ? Surely not Franz Liszt—he is the composer. It is Etude no. 5 . Because the subject of the main clause should also be the subject of the participial phrase, the correct choice is B.

When a participial phrase begins a sentence, its subject should be the subject of the main clause that follows. Otherwise, it is called a dangling participle .

What are participles , anyway?

Participles are verb forms, like broken and thinking , that cannot stand by themselves as verbs. They are only part of the verb, hence the name “participle.” Notice, for instance, that we can”t say

She broken the plate .

We thinking about you .

Each participle requires a helping verb to complete the verb phrase and make a sensible clause:

She has broken the plate .

We were thinking about you .

Present participles like eating , fighting , and interrupting always end in -ing . Past participles , however, fall under two categories: “regular” past participles like straightened and pushed end in -ed , but “irregular” past participles can take many forms, like fought , been , eaten , swum , and seen . For a list of some common irregular forms, see Lesson 25.

In English, we use present participles (with the helping verb to be ) in verbs with the progressive aspect (Lesson 23), such as I am eating and I had been eating . We use past participles (with the helping verb to have ) in verbs with the consequential aspect (Lesson 23) such as I have eaten and I had eaten .

When participles appear without their helping verbs, they act as adjectives, and their phrases are called participial phrases . Here are some more examples:

When designing a user interface , software engineers should focus on simplicity .

Although pleased with her victory , Angela knew that she still had more work to do .

Lesson 13: Know where to place your modifiers

Which is correct?

  1. In an emergency, I am amazed at how calm Marco can be.
  2. I am amazed at how calm Marco can be in an emergency.

What does the prepositional phrase in an emergency modify? It answers the question When can Marco be calm? rather than When can I be amazed? Since it modifies the second verb rather than the first verb, B is the better choice.

Any modifier or modifying phrase should be placed as close (or “proximate”) as possible to the word it modifies without disrupting the sentence. This is called the Law of Proximity . Modifiers or modifying phrases that violate this rule are called misplaced modifiers .

Which is correct?

  1. A splendid example of synthetic cubism, Picasso paintedThree Musicians in the summer of 1924 .
  2. Picasso paintedThree Musicians, a splendid example of synthetic cubism, in the summer of 1924 .

What does the appositive phrase a splendid example of synthetic cubism modify? It answers the question What is The Three Musicians? rather than Who was Picasso? Since it modifies the second noun, not the first, choice D is correct.

Which is correct?

  1. To illustrate his point, we watched Mr. Genovese take out a giant boa constrictor.
  2. We watched Mr. Genovese take out a giant boa constrictor to illustrate his point.

What does the infinitive phrase to illustrate his point modify? It answers the question Why did he take it out? rather than Why did we watch it? Since it modifies the second verb rather than the first, choice F is correct.

Exercise 6: Dangling and Misplaced Modifiers

Rewrite each underlined portion, if necessary, to correct any dangling or misplaced modifiers.

1 . Rounding the bend , the pub of my dreams finally came into view.

2 . Although emotionally drained, Martha”s creative instinct compelled her to keep writing.

3 . Determined to avenge his friend, the sword was unsheathed by Claudius .

4 . To find a good Thai restaurant, there are a lot of apps and websites to help you .

5 . Even with a sprained ankle, the coach forced Adam back into the game.

6 . We found my lost earrings walking back to my car .

7 . Lacking any real sailing skills, David”s primary concern was keeping the boat upright.

8 . Already exhausted from the day”s climb, the looming storm forced the hikers to pitch an early camp .

9 . Thinking that her friends were behind her, it frightened Allison to realize that she was alone.

10 . Without being aware of it , termites can infest your home unless you take proper precautions.

11 . Always regarded as a dutiful mother, we were surprised to hear Carol complaining about domestic life.

12 . To get a good jump out of the starting blocks, sprinters say that proper hip positioning is essential .

13 . Seeking ways to reduce the budget deficit, proposals for cutbacks are being considered by the town council .

14 . Although unhappy with the tone of the debate, the senator”s plan was to remain calm and rational.

15 . Famous for its visual arts scene, Portland”s musical culture is also a source of local pride.

16 . Without seeming to move a muscle, the coin disappeared instantly from the magician”s hand .

17 . To maintain good health, physicians recommend both vigorous exercise and disciplined eating .

18 . After searching for months for the perfect rug, one appeared as we were exploring a garage sale.

Lesson 14: Don”t confuse adjectives and adverbs

Which is correct?

  1. I was impressed by how poised he was and how cogent his argument was presented.
  2. I was impressed by how poised he was and how cogent his argument was.
  3. I was impressed by how poised he was and how cogently he presented his argument.

At first, reading, sentence A seems to follow the law of parallelism: it follows the formula I was impressed by A and B , and the phrases how poised and how cogent have the same form. However, the adjective in the second phrase is misused: we cannot say his argument was presented cogent , but rather his argument was presented cogently . Action verbs like presented can only be modified by adverbs , not adjectives . Sentence B corrects the modifier error but uses stilted phrasing. Sentence C, the best of the three, although less strictly parallel than sentence B, corrects the modifier error in A and the stiffness of sentence B.

Don”t use an adjective to do the job of an adverb. Many popular advertisements grab your attention by replacing adverbs with adjectives, as in Think different , Eat fresh , Shine bright , and Live strong . But in Standard English, adjectives are strictly noun modifiers . If you want to modify a verb (or an adjective or another adverb), only an adverb will do. Most adverbs end in -ly (as in profoundly , quickly , and desperately ), but many common ones do not.

Common adverbs that do NOT end in -ly :

always , away , ever , never , there , here , so , too , yet , very

Common adjectives that DO end in -ly :

lovely , lonely , motherly , neighborly , friendly , costly , beastly , lively , womanly , likely , scholarly

Common words that can serve EITHER as adjectives or adverbs :

If you have trouble deciding between using an adjective and using an adverb, ask: “What question does this word answer?” If it is a question about a noun or pronoun, the modifier must be an adjective. If it is a question about a verb, adjective, or another adverb, the modifier must be an adverb.

Lesson 15: Know when to use -er, -est, more , and most

Which is correct?

  1. I don”t know which is most troubling: your apathy or your incompetence.
  2. I don”t know which is more troubling: your apathy or your incompetence.

Sentence A is comparing only two things: apathy and incompetence , so it must use the comparative form, more , instead of most . Sentence B is correct.

If a sentence compares two things at a time (we call this a binary comparison), it must use a comparative adjective , that is, one that use -er or more . If the sentence singles out one thing from a group of three or more, it must use a superlative adjective , that is, one that uses -est or most .

Which is correct?

  1. Your dog couldn”t be adorabler.
  2. Your dog couldn”t be more adorable.

Which is correct?

  1. Incorporating the company was more simple than I expected.
  2. Incorporating the company was simpler than I expected.

When do we use -er , and when do we use more ? The rule is actually pretty straightforward.

If an adjective has just one or two syllables, it usually takes the -er suffix (e.g., faster , stronger , sillier ), but if it has more than two syllables, it usually takes more (e.g., more beautiful , more outrageous , more desperate ).

However, monosyllabic past participles, when used as adjectives, also tend to take more rather than -er : we say more set in his ways rather than setter in his ways, more shocked rather than shockeder , and more tired rather than tireder .

Fun is another interesting exception. Although something that is comparatively funny is funnier , something that is comparatively fun is more fun . For some reason, Standard English has decided against funner .

So, in the sample sentences, choices D and F are correct.

Which is correct?

  1. Please hold the baby gentler next time.
  2. Please hold the baby more gently next time.

Here, the problem with sentence G is the problem we discussed in Lesson 14: an adjective is being used where an adverb is required. Since the modifier is answering the question “How should one hold the baby?” it is answering a question about the verb hold , and therefore should take the adverbial form more gently .

Which is correct?

  1. Annie is the most unique person I know.
  2. Annie is unique.

The adjective unique is known as an “absolute” or “superlative” adjective. It comes from the Latin uni , meaning “one,” and it means “one of a kind.” Therefore, tacking on most is redundant. Sentence K makes the same point without the redundancy.

Don”t modify absolutes like perfect , unique , singular , or obliterated unless you are trying to be ironic.

Exercise 7: Using Modifiers Correctly

Correct any modifier problems in the sentences below.

1 . In the second debate, the councilwoman made her points much stronger than she did in the first one.

2 . My shirt smelled foully after rugby practice.

3 . We never usually get to go on such exotic vacations.

4 . My father is the most patient of my parents, but my mother is more knowledgeable about relationships.

5 . The sixth graders weren”t hardly interested in going to the museum after school.

6 . I can run a marathon easier than I can swim three miles.

7 . As you revise your essay, try to express your thoughts clearer and develop your ideas more.

8 . The chemistry final was much more easy than the last two chapter tests.

9 . Caroline”s sculpture was the most unique among the entries.

10 . These cost-cutting measures won”t barely address the budget deficit.

11 . The teacher never told us about the test until the day before.

12 . Students never usually verify the “facts” they use in their research papers.

Rule 6: Make Your Comparisons Clear and Precise

Lesson 16: Make sure your comparisons are logical

Which is correct?

  1. Not only is Anna the captain, but she also practices harder than anyone on the track team.
  2. Not only is Anna the captain, but she also practices harder than anyone else on the track team.

Anna cannot work harder than she herself does, and she is on the track team, so the first comparison is illogical . It is logical, however, to say that she works harder than anyone else on the track team , so sentence B is correct.

Which is correct?

  1. The turnout for this year”s art festival was even better than last year.
  2. The turnout for this year”s art festival was even better than the turnout for last year”s festival.

The phrase even better indicates a comparison, but between what two things? In sentence C, this year”s turnout is being compared to last year . This is another type of illogical comparison called a category error : the two things being compared are not comparable things. Sentence D corrects this error because the turnout for last year”s festival is in the same category as the turnout for this year”s festival . Since this is an “apples-to-apples” comparison, sentence D is correct.

Make sure all of your comparisons are logical comparisons .

  • Make sure that equivalent things are not treated as non-equivalent things. (For instance, Anna can”t practice harder than herself.)
  • Make sure that non-comparable things are not treated as comparable things (For instance, this year”s turnout can”t be compared to last year, but it can be compared to last year”s turnout.)

Lesson 17: Know how to use less/fewer, many/much , or amount/number

Which is best?

  1. To decrease the amount of violent conflicts among rival fans, the concession stands will sell less alcoholic drinks during the game.
  2. To decrease the number of violent conflicts among rival fans, the concession stands will sell fewer alcoholic drinks during the game.
  3. To decrease the amount of violence among rival fans, the concession stands will sell less alcohol during the game.

The terms less , much , and amount apply generally to uncountable or continuous quantities like traffic , money , and food . The terms fewer , many , and number apply generally to countable and discrete quantities like cars, dollars , and pizzas .

But what if the quantities are countable and continuous , like miles, gallons , or miles per gallon ? For instance, would you say This car gets fewer miles per gallon or This car gets less miles per gallon ? The answer depends on whether the context suggests you should emphasize the quantity”s countability (in which case you should use fewer ) or its continuity (in which case you should use less ). Of course, you could avoid the problem altogether by saying This car is less efficient .

Sentence A is problematic because it uses amount and less in reference to countable and discrete quantities, conflicts and alcoholic drinks . Sentence B corrects the problem by switching to number and fewer , but sentence C, which changes the quantities themselves to violence and alcohol , sounds more natural. The SAT will not expect you to choose between choices B and C on a multiple-choice question, because technically both are correct.

Exercise 8: Making Logical Comparisons

Correct any illogical comparisons in the sentences below.

1 . The show was universally praised by critics, who said consistently that it was more intelligent and provocative than anything on the air.

2 . Team unity and a strong work ethic were the key to their success.

3 . Mathematics lessons in Japanese classrooms, unlike American classrooms, are often focused on solving a single complex problem rather than many simplistic problems.

4 . The hybrid electric-combustion engines of the new cars are much quieter than conventional cars.

5 . To the critics of the time, the surrealists were regarded as being as inscrutable, if not more so, than the Dadaists.

6 . Modernist poetry was far less accessible to the readers of its time than was Shakespeare.

7 . Her suitcase would not close because she had packed too much of her towels into it.

8 . The year-end bonus was equally divided between Parker, Alyssa, and me.

9 . Many students wanted to be a lifeguard at the club.

10 . The toughest thing about her class is you have to do so much homework every night.

Rule 7: Make Sure Your Pronouns Are Clear and Precise

Lesson 18: Make sure your pronouns agree with their antecedents

Which is correct?

  1. Our financial team strictly maintains the confidentiality of their clients.
  2. Our financial team strictly maintains the confidentiality of its clients.
  3. Our financial counselors strictly maintain the confidentiality of their clients.

Every definite pronoun like it , him , herself , and their takes the place of a noun or pronoun called the antecedent . Every definite pronoun must agree in number (singular or plural) and category.

In sentence A, the definite pronoun, their , is plural, but the antecedent, team , is singular. This is a number disagreement. (At least it is in Standard American English; in Standard British English, collective nouns like team , crowd , and committee are treated as plurals.) Sentence B corrects this problem but introduces a subtle number shift and implies (probably incorrectly) that the entire team shares its clients. Sentence C corrects both problems and so is the best choice.

Which is correct?

  1. Sabrina, surprisingly, was the one that broke the silence.
  2. Sabrina, surprisingly, was the one who broke the silence.

Which is correct?

  1. The filibuster is a strategy where senators can extend debate in order to prevent a vote.
  2. The filibuster is a strategy in which senators can extend debate in order to prevent a vote.

Interrogative pronouns are the pronouns we use to ask questions, like who , what , where , and when . When these pronouns are not used to ask questions, they serve as definite pronouns that refer to the nouns that immediately precede them (that is, they serve as appositives ). Like all definite pronouns, they must agree in category with their antecedents.

The pronouns in sentence D and sentence F both disagree in category with their antecedents: Sabrina is a person, not a thing, so who is a more appropriate pronoun than that . The filibuster is a procedure, not a place, so which is a more appropriate pronoun than where .

Lesson 19: Avoid ambiguous pronouns

What is wrong with the following sentences?

  1. The coach told Mike that he was going to miss the next game.
  2. The main difference between scientific thinking and ideological thinking is that it gives evidence priority over belief.

Both of these sentences are ambiguous. In sentence A, who will miss the game, Mike or the coach ? In sentence B, which way of thinking gives evidence priority, scientific thinking or ideological thinking ? Both sentences should be revised to eliminate ambiguous pronouns .

  1. The coach said that he would bench Mike for the next game.
  2. Scientific thinking, unlike ideological thinking, gives evidence priority over belief.

Lesson 20: Maintain consistency with your pronouns

Which is correct?

  1. My wife and I enjoy attending our school reunions because you meet so many interesting people there.
  2. My wife and I enjoy attending our school reunions because we meet so many interesting people there.

The pronoun references in sentence A are inconsistent: the generic pronoun you conflicts with the personal explanation indicated by the context, so the use of we in sentence B is more appropriate.

Which is correct?

  1. The flying squirrel uses its patagium—a membrane extending from the wrist to the ankle—as a parachute to help them glide safely out of the reach of predators.
  2. The flying squirrel uses its patagium—a membrane extending from the wrist to the ankle—as a parachute to help it glide safely out of the reach of predators.

Sentence C commits a pronoun shift . The pronoun referring to the flying squirrel has shifted from its to them . Sentence D makes the correction.

Watch your pronouns to make sure that they don”t shift . Once you choose a pronoun to refer to a particular antecedent, stick with it.

Exercise 9: Using Pronouns

Circle all pronouns and rewrite to correct any pronoun errors.

1 . This is one of those times in a game where an undisciplined player can lose focus or forget about strategy.

2 . If a student wants to learn the meaning of a word, begin by learning its relevant context.

3 . Caroline passed the phone to Julia, but she couldn”t bring herself to speak.

4 . Not wanting to be the one that slowed the team down, David dropped out of the race.

5 . Brown is committed to assisting their students by providing him or her with any necessary financial aid.

6 . The media ignored the reports because it didn”t consider them newsworthy.

7 . No one that has been through the first week of boot camp ever believes that they will make it through the entire six weeks.

8 . Although one should never read carelessly, you should move briskly through the page to maintain focus on the purpose behind the text.

9 . Neither Jack nor Ted thought that their team could lose the game, even when he began missing his shots.

10 . Students sometimes aren”t ready to handle the extra work that is required when his or her courses become more demanding.

11 . I enjoy reading stories where underdogs eventually triumph.

12 . Everyone will be expected to do their share to prepare the camp for visitor”s day.

13 . The museum received so many donations that they surpassed their fund-raising goal for the year.

14 . The judges usually give the trophy to the performer that makes the fewest mistakes.

15 . We have configured the pool so that each swimmer will have a lane to themselves.

16 . Who was the player that hit the home run?

Lesson 21: Use the correct pronoun case

Each of these sentences contains one pronoun error. Can you find it?

  1. As the waiter was talking to Jenna and I, he showed us the tattoo on his neck.
  2. I don”t know anyone who can run a campaign more effectively than her.
  3. Although Carl said he wasn”t hungry, the first one at the buffet was him.
  4. The team voted and selected myself as the next captain.

These pronoun errors are called errors in case . Here are the corrections:

  1. As the waiter was talking to Jenna and me, he showed us the tattoo on his neck.
  2. I don”t know anyone who can run a campaign more effectively than she can.
  3. Although Carl said he wasn”t hungry, he was the first one at the buffet.
  4. The team voted and selected me as the next captain.

The case of a pronoun refers to its relationship to the verb. If a pronoun serves as or is equated with the subject of a verb, it takes the subjective case . If it serves as the direct or indirect object of the verb, it takes that objective case . If the object of the verb has the same referent as the subject , then it takes the reflexive case . If it indicates possession, it takes the possessive case .

In sentence A, the pronoun I is the object of the prepositional phrase to Jenna and I , and so it requires the objective case, as in sentence E. In sentence B, the comparative phrase more effectively is adverbial, indicating that the comparison is between verbs in the clauses who can run and she [can run] , so the pronoun her should be changed to the subjective case, as in sentence F. In sentence C, the verb was is a linking verb , which means that the pronoun him is being “equated” with the subject one , and therefore should be changed to the subjective case, as in sentence G. (Notice, also, that sentence G “inverts” the main clause from sentence C so that it is parallel with the first clause.) Sentence D abuses the reflexive case , which is the subject of our next lesson.

Lesson 22: Don”t abuse reflexive pronouns

Which is correct?

  1. Either Caroline or myself will open the account this week.
  2. Either Caroline or I will open the account this week.

A reflective pronoun should only be used as

  • the object of a verb when it is identical to the subject:

e.g., I did it all by myself. She cut herself .

  • an emphaticappositive (Lesson 13): I myself would never do such a thing .

Do NOT use reflexive pronouns as ordinary subjects or objects.

Since myself is part of the subject phrase, it must take the subjective case; therefore sentence B is correct.

Exercise 10: Pronoun Case

Circle the correct pronoun in each sentence.

1 . The climb was much easier for Camille than it was for Jeff and (I/me/myself).

2 . The other contestants did not seem as confident as (he/him/himself).

3 . (Us/We) detectives are always careful to follow every lead.

4 . Every student should make (his or her/their) own study plan.

5 . The administrators never seem to listen to the opinions of (us/we) students.

6 . Jim gave control of the project to Fiona and (me/myself/I).

7 . The university presented the honor to David and (he/him/himself).

8 . Justine and (me/I/myself) have always been closest friends.

9 . There is no point in (our/us) delaying the tests any longer.

10 . It seems quite clear that you and (I/me) will have to work together to solve this problem.

11 . It might be difficult for (him and me/he and I) to agree on a topic.

12 . (We/Us) and the other new members debated the issue for over two hours.

13 . The owners of the club offered my wife and (I/me/myself) a free bottle of wine with dinner.

14 . No other member of the team could outrun (I/me/myself).

15 . The teachers were getting tired of (him/his) constantly falling asleep in class.

16 . Major League ballparks have always held a special attraction for Dave and (I/me).

17 . I am concerned about (you/your) taking so much time off work.

Rule 8: Make Your Verbs Clear and Precise

Lesson 23: Know how to use the consequential or “perfect” aspect

Which is correct?

  1. It doesn”t really matter now, because I have been to the mountaintop.
  2. It doesn”t really matter now, because I was on the mountaintop.
  3. It doesn”t really matter now, because I went to the mountaintop.

Why do sentences B and C sound so uninspiring in comparison to sentence A (adapted from Martin Luther King Jr.”s last speech)? They fall flat because they destroy the meaning conveyed by the tense and aspect of the verb in sentence A. The tense of a verb indicates its place in time: past, present, or future, but the aspect of a verb indicates how its action or status extends to the subject.

Sentence A is obviously about who King is now as a consequence of a previous event , not simply about what he did in the past . In other words, it requires the present tense and the consequential (or “perfect”) aspect . Sentences B and C destroy this essential meaning by putting the verb in the simple past tense .

The aspect of a verb indicates how its action or status extends to the subject, and is generally independent of tense. For instance, a present tense verb can have many different aspects:

Grammatical forms of the consequential (or “perfect”) aspect:

Use the consequential (or “perfect”) aspect (e.g., have taken, had taken, will have taken ) when you want to indicate that a status is the consequence of a previous action or status .

Lesson 24: Know how to express historical facts and general ideas

Which is correct?

  1. In his bookWalden, Thoreau provided a manifesto for self-reliance .
  2. In his bookWalden, Thoreau provides a manifesto for self-reliance .

Which is correct?

  1. The ancient Greek philosopher Zenotaught that motion was an illusion .
  2. The ancient Greek philosopher Zenotaught that motion is an illusion .
  3. The ancient Greek philosopher Zenoteaches that motion was an illusion .
  4. The ancient Greek philosopher Zenoteaches that motion is an illusion .

Because both Zeno and Thoreau are long dead, the first version of each sentence, with past tense verbs, may seem correct. However, it is important to ask: do these sentences indicate historical facts or general ideas ?

In Standard English, historical facts take the past tense, but statements about general ideas and references to the content of widely available artistic works usually take the present tense. In an ambiguous case, such as when referring to an idea that has been refuted over the course of history, choose the tense that emphasizes the appropriate quality: use the present tense if you intend to emphasize its “idea-ness,” but use the past tense if you intend to emphasize the fact that it is “history.”

For the first pair of sentences, context is everything. If the sentence were part of a paragraph discussing Thoreau”s life or the history of Transcendentalism, it would be a statement of historical fact, and so choice A would be preferred. If, however, this sentence were part of a discussion of the ideas in Walden , then sentence B would be correct.

The second sentence includes two clauses. The first refers to the historical fact that Zeno was a teacher, and the second refers to a general idea about motion. If you wish to emphasize the “idea-ness” of the second clause, then sentence D is the best choice. If you wish to emphasize the fact that this claim is “history” (that is, no longer believed), then sentence C is the best choice.

Lesson 25: Watch for irregular verbs

Which is correct?

  1. Peter was in pain after the run because he hadtore his Achilles tendon .
  2. Peter was in pain after the run because he hadtorn his Achilles tendon .

The verb in the second clause takes the consequential (or “perfect”) aspect (Lesson 24), which requires the past participle torn , not tore . The verb to tear is an irregular verb , which means that its past participle is not an -ed form of the verb. The correct sentence is B.

Here is a list of some common irregular verbs. Remember that verbs in the consequential or “perfect” aspect require the past participle form , not the past tense form . For instance, I have drank is the wrong form; I have drunkis correct.

Exercise 11: Verb Tenses and Aspects

Circle the verb form(s) that make each sentence coherent.

1 . This morning, Ryan (came/has come/comes) to work with bags under his eyes because he (stayed/had stayed/was staying) up all last night.

2 . Already, and without (spending/having spent) so much as an hour on research, Dale (wrote/has written/will write) the first draft of her essay.

3 . (Developing/Having developed) the first hydrogen cell automobile, the team (hoped/had hoped) to reveal it to the world at the technology exposition.

4 . Right after school, we (went/had gone) to Mario”s for pizza.

5 . Surprisingly, Catcher in the Rye (is/was/would be) the only full-length novel that the late J. D. Salinger ever (has published/published/will have published).

6 . (Finding/Having found) no evidence against the accused, the detectives (had/had had) to release him.

7 . (Being/Having been) captured by the rebels, David soon (began/had begun) to fear he would never escape.

8 . When I (arrived/had arrived) home from the museum, I (started/had started/will start) to plan my art project.

9 . By the time the committee (adjourned/had adjourned), it (voted/had voted) on all four key proposals.

10 . As the seventh inning stretch began, we (did not score/had not scored) a single run.

11 . In To Kill a Mockingbird , Harper Lee (uses/used/has used) the character of Dill Harris, whom she (bases/based/has based) on her real-life friend Truman Capote, to embody youthful innocence and imagination.

12 . That evening, we (had/had had) a lovely meal with the group with whom we (hiked/had hiked) all afternoon.

13 . (Walking/Having walked) all night, this morning we (were/had been) desperate to find a resting spot.

14 . By the time I am done with finals, I (will write/will have written) four major papers.

15 . (Winning/Having won) her previous three races, Anna (was/had been) confident that she (will win/would win) the next one as well.

16 . It surprised us to learn that Venus (is/was/had been) almost the same size as Earth.

17 . Buyers often (worry/have worried/will worry) too much about finding a low mortgage rate, and (forget/have forgotten/will forget) to scrutinize the terms of the contract.

18 . I am qualified for this job because I (completed/have completed/had completed) two courses in digital marketing.

19 . During the time of the ancient Greeks, many physicians (believed/had believed) that illnesses (are caused/were caused) by imbalances in bodily fluids.

20 . Students (often worry/will often worry) excessively about grades and not enough about understanding.

Rule 9: Make the Rest of Your Sentence Clear and Precise

Lesson 26: Avoid redundancy

Which is correct?

  1. With only seconds remaining left to go in the game, Michael grabbed the ball and sped quickly down the court.
  2. With only seconds to go in the game, Michael grabbed the ball and sped down the court.

Notice that sentence A does not convey any idea that is not also conveyed in sentence B. Therefore, the three words that have been removed are redundant . Sentence B is better because it obeys the Law of Parsimony.

The Law of Parsimony

All else being equal, shorter is better.

Only one of remaining , left , or to go is necessary, because they all have the same meaning. Also, since sped means moved quickly , the adverb quickly is redundant.

Lesson 27: Avoid diction errors

Which sentence is best?

  1. The news about the court”s ruling extended quickly throughout the Internet.
  2. The news about the court”s ruling scattered quickly throughout the Internet.
  3. The news about the court”s ruling propagated quickly throughout the Internet.
  4. The news about the court”s ruling expanded quickly throughout the Internet.

None of these sentences is grammatically wrong, but sentence A sounds odd. The word extended is not quite right for this context. From the Latin tendere which means “to stretch,” extend applies to things, like baseball games or necks, that are made to go beyond their typical lengths. But news , unlike a baseball game or a neck, does not have a “typical length,” so trying to apply the verb extend to it is a diction error : the inappropriate use of a word.

Sentence B sounds a bit better, but scatter applies to a bunch of individual things, like seeds or mice, that are suddenly moving away from their group. Since this news is a single fact, not many individual items in a bunch, scattered doesn”t quite work, either.

Sentence C uses propagated , which means spread or promoted, as an idea or theory . Since news spreads very much as an idea or theory does, the verb is being used appropriately.

Sentence D uses expanded , which, like extended , typically refers to something growing beyond its typical size or limit. Since news doesn”t have a typical size or limit, expanded is not quite the right word.

Which sentence is correct?

  1. We interviewed about thirty perspective candidates for the job.
  2. We interviewed about thirty prospective candidates for the job.

The diction error in sentence E is a “sound-alike ” error. The word perspective is a noun meaning “point of view,” but the sentence clearly calls for an adjective describing the candidates. Prospective is an adjective meaning “expected to play a particular role or to achieve a particular goal in the future,” which is certainly appropriate in describing a job candidate.

Common “sound-alikes”

accept (v) = to agree to take < accept an offer >

except (prep) = not including <every day except Sunday>

except (v) = exclude <present company excepted >

adapt (v) = to make suitable for a particular purpose < adapted to a new use >

adopt (v) = to choose as one”s own < adopt a child >

adept (adj) = highly skilled <an adept player >

affect (v) = to influence <it affected me deeply >

effect (n) = result or consequence <had a good effect >

allude (v) = to make a subtle or indirect reference (to) <he alluded to their first meeting >

elude (v) = to escape from; to avoid < elude capture >

allusion (n) = a subtle reference <an allusion to Othello>

illusion (n) = misconception or misperception <optical illusion >

ambivalent (adj) = having conflicting feelings (about) <I feel ambivalent about going to the party >

ambiguous (adj) = unclear or having more than one interpretation <an ambiguous signal >

cite (v) = to credit as a source of information < cite an article >; to commend for meritorious action < cited for bravery >

site (n) = location where a particular activity occurs <the site of the battle >

sight (v) = to see at a specific location <she was sighted in the crowd >

compliment (n) = a praising personal comment < compliments are always appreciated>

complement (n) = something that completes or makes a whole <Brie is a fine complement to this wine >

council (n) = a committee <the executive council >

counsel (v) = to give advice <he counseled me wisely >

discrete (adj) = distinct <dozens of discrete parts >

discreet (adj) = prudently modest in revealing information <please be discreet about our meeting >

elicit (v) = to bring out or to call forth <the joke elicited uncomfortable laughter >

illicit (adj) = unlawful < illicit activities >

eminent (adj) = prominent and distinguished <an eminent historian >

imminent (adj) = about to happen < imminent doom >

flaunt (v) = to show (something) off <if you”ve got it , flaunt it >

flout (v) = to show disregard for < flout the rules >

gambit (n) = a careful strategy or an opening move <a bold gambit >

gamut (n) = the complete range <run the gamut >

imply (v) = to suggest or hint at <a handshake implies agreement >

infer (v) = to draw a conclusion from evidence <we can infer hostile intent >

morale (n) (mor-AL) = shared enthusiasm for and dedication to a goal <the team”s morale was high >

moral (n) (MOR-al) = lesson or principle about good behavior <the story had a nice moral >

phase (n) = stage in a process <third phase of the project >

faze (n) = to disturb (someone”s) composure < fazed by the interruption >

precede (v) = to come before <thunder is always preceded by lightning >

proceed (v) (pro-CEED) = to go on, usually after a pause (pro - forward) < proceed with the task >

proceeds (n) (PRO-ceeds) = funds received from a venture < proceeds from the raffle >

principal (n) = head of a school < principal Skinner is well liked> ; the initial investment in an interest-bearing account <many investments risk a loss of principal >

principle (n) = guiding rule <the principle of the matter >

reticent (adj) = reserved or reluctant to talk freely <she has been reticent in therapy>

reluctant (adj) = disinclined to do something < reluctant to reveal personal information >

Exercise 12: Diction Problems

Choose the best word in the sentences below.

1 . Even the most trivial news seems to (affect/effect) the stock price immediately.

2 . Even the most aggressive pesticides could not (delete/remove/eradicate/abolish) the beetles.

3 . The (moral/morale) of the troops was at an all-time low during the Christmas season.

4 . That scarf really (compliments/complements) your outfit.

5 . Many well-trained oenologists can (separate/distinguish/acknowledge/certify) the tastes of dozens of different grapes.

6 . The article emphasized the low voter turnout in order to (imply/infer) that the senator may not have been elected by a true majority.

7 . The justices can debate a case for weeks before a formal ruling is (appointed/specified/chosen/predetermined/given/designated).

8 . It may be years before we understand how pollution from the new power plant might (affect/effect) the regional environment.

9 . The negotiations became very (apprehensive/tense/neurotic/fretful/anxious) when the topic of old tribal conflicts was broached.

10 . Heather was the (principal/principle) author of the study that was recently published in a prominent scientific magazine.

11 . Although enormously popular among filmgoers, the movie was soundly (disparaged/confronted/molested/eradicated/charged/impaired) by critics.

12 . The words and images in advertisements are carefully chosen to subtly (propel/compel/extort/oppress/oblige) consumers into buying things they may not want.

13 . Try as they might, the hikers could not find the (antidote/anecdote) to the snake venom.

14 . The acid solution was so potent that we had to (dilute/delude) it with water before we could use it safely.

15 . Annie”s project (excelled/overshadowed/outstripped/exceeded/preceded) all of our expectations.

16 . Originally built for a small tractor, the engine had to be (correlated/attuned/converted/reoriented/improved) for use as a boat motor.

17 . As someone committed to fairness in education, she could not accept the (iniquity/inequity) of the admissions policy.

18 . Although most of the manuscripts were signed by their authors, some were written (anonymously/unanimously).

19 . It was hard for the comic to (elicit/illicit) even the slightest laugh from the crowd.

20 . We needed to (adapt/adopt/adept) the play to make it appropriate for younger audiences.

21 . Darryl”s self-esteem (enlarged/blossomed/multiplied/escalated/proliferated) once she found a peer group that shared her interests.

22 . She thought she should be (discreet/discrete) about their relationship.

23 . The (council/counsel) will decide how to finance the new city park.

24 . Rather than obeying the coach, Richard always tries to (flaunt/flout) the team rules.

25 . His knowledge of sports runs the (gamut/gambit) from table tennis to arena football.

26 . The jury should not (infer/imply) guilt from the defendant”s refusal to answer these questions.

27 . The builders had to (truncate/curtail/lower/belittle/subside) their work during the evening hours after the neighbors filed a complaint.

28 . Rather than eliminate the department all at once, they decided to (faze/phase) it out gradually.

29 . Barking dogs can often signal (imminent/eminent) danger.

30 . After our vacation, we decided to (proceed/precede) with the plan.

31 . Recent diplomatic efforts have focused on (defusing/declining/dwindling/degrading/discounting) the conflict by promoting nonconfrontational dialogue of all sorts.

32 . I always felt (reticent/reluctant) to talk in class.

33 . The democratically elected government has been forcefully (shifted/substituted/exchanged/supplanted) by a military cabal.

34 . The police officer was (cited/sighted) for her efforts in the hostage rescue.

Eliminate any redundant words or phrases in the paragraph below.

35 . When we look back to past history, we see that whenever a new innovation is introduced for the first time, people rarely accept the whole entire concept, at least not right away. If and when something threatens the ways of the past, people don”t easily accept this new concept. Societies necessarily need stability because consistency and predictability make people feel comfortable and minimize conflict. Even when technology gives us a more efficient method, we often continue on with our older, less efficient ways. For instance, it”s not uncommon to see people using e-mail for quick communications while at the same time they could have just texted to accomplish the same thing. If we take a moment to pause and consider for a second, it doesn”t take much to see we can see that we can communicate more efficiently by text. And there are even some traditionalists who like the old way of doing things and will write letters on paper, which requires killing trees!

Lesson 28: Avoid errors in idiom

What is the difference between these two sentences?

  1. If you want to make friends, you should go on in the party.
  2. If you want to make friends, you should go in on the party.

These sentences use different semantic idioms , and so give very different advice. When you tell someone to go on in , you are giving him or her casual permission to enter, so sentence A says that casually inserting yourself into a social situation can make you more likeable. When you ask someone to go in on something, you are asking him or her to contribute money to the effort, so sentence B says that the folks throwing the party would like you more if you kicked in a few bucks. A semantic idiom is a common phrase with an established meaning, like push through , on fire, see the light , or go in on , that differs from its literal meaning.

Errors in idiom are usually “wrong preposition” errors. In some idiomatic phrases, the choice of preposition is essential to the meaning: for instance, breaking up , breaking down , breaking in , and breaking out are all very different activities. In other idiomatic phrases, such as the standard parallel constructions described in Lesson 10, the preposition is simply a matter of convention. For instance, the sentence Thai food is very different than Cantonese food contains an error in syntactical idiom. The preposition than should only be used with comparative adjectives, as in smaller than , faster than , and harder than . But different is not a comparative adjective and instead takes the preposition from . We should say Thai food is very different from Cantonese food .

Which is correct?

  1. Effective therapy depends both on consistent adherence to the protocol as well as regular recalibration of the medication dosage.
  2. Effective therapy depends both on consistent adherence to the protocol and regular recalibration of the medication dosage.
  3. Effective therapy depends on both consistent adherence to the protocol and regular recalibration of the medication dosage.

Sentence C uses the word both , which can either be followed by a simple plural noun (both legs , both kinds ) or a prepositional phrase (both of them ) or be part of a standard parallel construction , both X and Y , which we saw in Lesson 10. A standard parallel construction is a syntactical idiom , that is, a rigid way of phrasing relationships between ideas. Notice that the phrasing in sentence C—both X as well as Y— is nonidiomatic . The phrasing in D is idiomatic but nonparallel (Lesson 9): X is a prepositional phrase but Y is a noun phrase. Sentence E is both idiomatic and parallel, and is the best choice.

When writing formally, remember to ESP: eliminate superfluous prepositions . We often use “extra” prepositions in informal speech, such as the redundant prepositions in climb up , fall down , and fight against . Notice how eliminating the unnecessary prepositions in these sentences makes them sound more “proper”:

Her superior skill and strength helped her to dominate over her opponents .

Many of our neighbors helped out with the renovation of the old firehouse .

You don”t want to miss out on all the fun .

Their attempt to extract out the harmful chemicals was unsuccessful .

Exercise 13: Errors in Idiom

Choose the correct preposition, or “none” if none is required.

1 . I prefer the soft light of an incandescent bulb (to/over/more than/none ) the harsh light of some fluorescent bulbs.

2 . We all agreed (on/with/about/none ) a plan to go skiing rather than hiking.

3 . The defendant would not agree (to/on/with/about) the plea bargain.

4 . We found dozens of old photographs hidden (in/none ) between the pages.

5 . Good study habits are necessary (to/for/in/none ) academic success.

6 . The new house color is not very different (from/than/to/none ) the old one.

7 . Margot was angry (with/about/at/none ) Brian for not telling her that he was leaving.

8 . They were both angry (about/at/with/none ) the boys” behavior.

9 . A lawyer should review the contract to see that it complies (with/in/about/to/none ) the laws of your state.

10 . The interview provided insight (about/into/for/none ) the creative process of great directors.

11 . We were very angry (about/with/at/against/none ) him for ignoring our phone calls.

12 . We all agreed (with/on/to/about/none ) the high quality of the food.

13 . Her tests include questions that seem very different (than/from/of/none ) those that we see in the homework.

14 . When she arrived on campus, she felt truly independent (of/from/none ) her parents for the first time.

15 . We were very angry (about/at/with/none ) the exorbitant price of gasoline at the corner gas station.

16 . It was hard not to agree (to/about/with/none ) her offer of a free evening of babysitting.

17 . I arrived at the meeting too late to raise my objection (against/to/of/none ) the proposal.

18 . If we don”t act soon, we may miss (out on/none ) the opportunity to lock in the lowest rates.

Lesson 29: Know how to use the active and passive voices

Which is better?

  1. I broke the paddle.
  2. The paddle was broken by me.

Sentence A and sentence B make the same statement, but in different voices : sentence A uses the active voice and sentence B uses the passive voice . In the active voice, the subject is the “actor” of the action, but in the passive voice, it is not.

For most declarative statements in which the actor is known, the active voice (e.g., I kicked the ball ) is clearer and more direct than the passive voice (e.g., The ball was kicked by me ).

Which is better?

  1. Henry ate all of his steak, but his vegetables were uneaten.
  2. Henry ate all of his steak but none of his vegetables.

In sentence C, the first clause is active, but the second is passive. This is not only a violation of the Law of Parallelism (Lesson 9), but also a subtle evasion: who failed to eat the vegetables? Sentence D is more parallel, clear, and direct.

Overusing the passive voice not only makes your sentences wordier, but also often indicates evasiveness , because the passive voice does not require the actor. For instance, a statement like I made a mistake cannot be construed as an evasion of responsibility when phrased in the active voice. However, the passive voice form A mistake was made by me , when “trimmed” (Lesson 3) becomes A mistake was made , which is clearly evasive.

Which is better?

  1. Although we enjoyed the hike to the peak, on the way down mosquitoes bit us, a thunderstorm drenched us, and countless thorns scratched us.
  2. Although we enjoyed the hike to the peak, on the way down we were bitten by mosquitoes, drenched by a thunderstorm, and scratched by countless thorns.

In sentence E, all three clauses at the end of the sentence are parallel and active, yet the sentence sounds strange. In sentence F introducing the passive voice improves the sentence by creating another level of parallelism, because now all four clauses have the same subject: we enjoyed … we were bitten … [we were] drenched … [we were] scratched .

Sometimes parallel structure requires using the passive voice.

Lesson 30: Understand your moods

Which is correct?

  1. If I was more patient, I would become a good violinist.
  2. If I were more patient, I can become a good violinist.
  3. If I were more patient, I could become a good violinist.

These sentences are conditionals , which take the form “If X, then Y” or simply “If X, Y” where X is a clause called the hypothesis and Y is a clause called the conclusion . The hypothesis takes different forms depending on whether it is occasional , unlikely , or counterfactual . The hypothesis here is unlikely or wishful and the conclusion indicates a possibility , so, as our discussion below will clarify, only sentence C has the correct form.

If the hypothesis is occasional or likely , then it takes the indicative mood ; that is, it is stated as a fact. For instance, theorems in mathematics and logic and statements about common consequences take this form:

If two sides of a triangle are congruent, then the two base angles are also congruent .

If I eat too much, I will have a hard time sleeping .

If you turn the switch, the light will go on .

If the hypothesis is present counterfactual , that is, it is unlikely or wishful, then it takes the present subjunctive mood. (Notice that a present subjunctive hypothesis, if it does not use the verb to be , can take the same form as the simple past tense .)

If I had a million dollars, I would buy a new house .

If Kate could tolerate the noise, she would come to the club with us .

If I were taller, I would play in the NBA .

If the hypothesis is past counterfactual , that is, it contradicts a state or event in the past, then it takes the past subjunctive mood. (Notice that a counterfactual hypothesis takes the same form as the past consequential , and the counterfactual conclusion takes the consequential aspect (Lesson 23).)

If I had caught the ball, we would have won the game .

If I had been more studious in college, I could have graduated cum laude .

Counterfactuals can also include indirect commands, wishes, expressions of doubt, hypothetical consequences, and suggestions, all of which take the subjunctive mood .

A mood is a verb category that indicates whether a clause is a factual statement (indicative mood , as in I went to the park ), a direct command (imperative mood , as in Go to the park! ), a question (interrogative mood , as in Did you go to the park? ), or a counterfactual (subjunctive mood , as in I should have gone to the park ).

Verbs that are in the subjunctive mood often require a subjunctive auxiliary , otherwise known as a “verb modal.”

The verb to be can sometimes take its subjective form without an auxiliary:

Which is correct?

  1. If we would have left earlier, we would not have been caught the storm.
  2. If we had left earlier, we would not have been caught the storm.

Again, sentence A is a conditional with a counterfactual hypothesis, indicating that a nonfactual condition would have a particular result. However, the auxiliary would indicates a conditional conclusion, not a conditional hypothesis. The counterfactual hypothesis takes the same form as the past consequential (Lesson 23), had left , as in sentence B.

Exercise 14: Mood and Voice

Circle the correct verb form in each of the following sentences.

1 . If our wide receiver (was/were) a little faster, he would get more open in the secondary.

2 . As a matter of fact, Theo (was/would have been) only six years old when the Civil War (had begun/began).

3 . Denny would be more successful if only he (promoted/would promote) himself more aggressively.

4 . The brochure suggested that we (are/be/would be) at the camp first thing in the morning.

5 . I wish that my horse (were/was) not so lethargic this morning.

6 . If the goalie (would have/had) lifted his glove even slightly, the puck (would have gotten/would get) through.

7 . He acted as though the concert hall (was/were) filled with screaming fans.

8 . I wish that summer camp (was/were) two weeks longer.

9 . If the class (would have/had) voted against it, we would not have purchased the new gerbil cage.

10 . We doubted that Joanna (will/would/might) get the part, since she was sick during her audition.

11 . If I (were/was/had been) in Paris, I would probably be spending most of my time at the Louvre .

12 . If I (might have/would have/had) known that the food was so good here, I (would have come/would come/came) sooner.

13 . The coach demanded that we (would be/be/should be/were) in bed by eleven o”clock.

14 . Yvonne acted as if she (was/were) the only customer in the restaurant.

15 . Gina wished that she (had/would have/will have) chosen the red dress instead of the pink one.

16 . The professor spoke to us as if he (was/were) an ancient Athenian general.

17 . I (would have wanted/wanted) to (have seen/see) the countryside, but I was sick in bed for the entire vacation.

18 . Had I found his wallet, I (would have/had/will have) returned it to him immediately.

19 . If only the doctor (had/would have) told me to cut back on eating red meat, I (would have/should have) complied.

Rule 10: Know How to Punctuate

Lesson 31: Know how to use apostrophes

Which is correct?

  1. Its hard to know when you”re dog has reached the limit of it”s stamina if your not checking it regularly during your run.
  2. It”s hard to know when your dog has reached the limit of its stamina if you”re not checking it regularly during your run.
  3. It”s hard to know when you”re dog has reached the limit of it”s stamina if you”re not checking it regularly during you”re run.

Apostrophes serve two main functions: to indicate missing letters in a contraction as in can”t (from cannot ), and to indicate possession , as in we went to Jacob”s house .

When turning a singular noun into a possessive adjective, simply add ”s , as in the committee”s decision . If the noun is a plural ending in , simply add an apostrophe, as in the sisters” relationship .

Several common contractions are homophones (sound-alikes) of possessives, and so the two are commonly confused. Fortunately, there is a simple rule to keep them straight: the contraction always gets the apostrophe:

Notice that sentence B above is the only one of the three that uses apostrophes correctly and avoids the its/it”s and your/you”re confusion.

Lesson 32: Know how to use commas

What is wrong with these sentences?

  1. The subject that intimidates me the most, is calculus.
  2. I could not help Justine with her project, I had just begun a new job.
  3. As we passed through Springfield, Massachusetts we stopped at the Basketball Hall of Fame.
  4. We will be discussing my favorite poem, “Leaves of Grass,” next semester .
  5. I would like to thank my parents, God and Ayn Rand.

Sentence A suffers from the stray comma syndrome . Simply put, the comma doesn”t belong. Chuck it.

The primary job of the comma is as a separator. It is used to separate

  • items in a list (e.g.,He was fat, dumb, and lazy .)
  • coordinate adjectives (e.g.,She gave a droning, uninspired speech .)
  • modifying phrases from the main clause (e.g.,In summary, I am appalled .)
  • dependent clauses that precede independent clauses (e.g.,Whenever I try, I fail .)
  • (with a conjunction) independent clauses from other independent clauses (e.g.,I think, therefore I am .)

It can also be used to

  • introduce a quotation (e.g.,Tom said, “I ain”t goin” where I ain”t needed.” )
  • format an address or date (e.g.,Saturday, July 19, 2014; Cleveland, Ohio )
  • to signal an addressee in dialogue or colloquial prose (e.g.,Get going, buster! )

Sentence B commits a comma splice (Lesson 6). Two independent clauses cannot be joined with just a comma. Either change the comma to a colon or semicolon, or insert a conjunction:

I could not help Justine with her project, because I had just begun a new job .

Sentence C omits the comma after the state name. It should read

As we passed through Springfield, Massachusetts, we stopped at the Basketball Hall of Fame .

Notice that this treats Massachusetts as an interrupter (Lesson 3), which is fine because the sentence reads correctly even when it is omitted.

Substantial modifying phrases in the middle of a sentence are called interrupting modifiers (Lesson 3) and should be separated from the main clause by commas. Remember that a sentence should read properly even when the interrupters have been removed.

In sentence D, the title of the poem works the same way as the state name in sentence C. It is a specifying modifier and requires commas before and after:

We will be discussing my favorite poem, “Leaves of Grass,” next semester .

When a comma follows a title or phrase in quotes, the comma must precede the end quote.

Sentence E omits the serial comma , the comma that separates the second-to-last item in a list from the conjunction and . The serial comma is almost universally accepted as proper and necessary in Standard American English, because without it sentence E becomes absurd. In this apocryphal dedication of a book, the lack of a serial comma makes it seem like the author believes she is the offspring of a deity and a childless woman. Of course, the author intends her dedication as a list of four, not two:

I would like to thank my parents, God, and Ayn Rand .

Two notable authorities that do not accept this rule are the New York Times and the AP (Associated Press) Stylebook , which recommend against the Oxford comma except to prevent an ambiguity such as that in sentence E.

The use of the serial comma (the second comma in the phrase A, B, and C ) in Standard American Usage is still a matter of debate and therefore will almost certainly not be tested on the SAT.

Lesson 33: Know how to use dashes

What is wrong with this sentence?

  1. The best that they could do—at least without a splint, was to set the broken bone and wait for help to arrive.

The dash (or, as it is sometimes known, the em dash ) is used to insert an abrupt break in thought in the middle or at the end of a sentence. If the break comes in the middle, then two dashes signify the beginning and the end of the interruption. In this case, the end of the interruption is indicated by a comma, where it should be a dash:

The best that they could do—at least without a splint—was to set the broken bone and wait for help to arrive .

If the interruption is not much of a departure from the main idea, then commas will work also:

The best that they could do, at least without a splint, was to set the broken bone and wait for help to arrive .

The punctuation on the two sides of an interrupter must be identical: either both em dashes or both commas.

Exercise 15: Punctuation

Correct any errors in punctuation (apostrophes, commas, dashes, colons, and semicolons) in the following sentences.

1 . Truman Capote”s nonfiction book, In Cold Blood is considered the first, greatest true crime novel.

2 . I could not see clearly, until my eyes adjusted to the bright lights.

3 . Runners, who step out of they”re lanes during the first lap, will be disqualified.

4 . Contrary to popular belief water will reach it”s boiling point more slowly, when its under greater pressure.

5 . In my opinion the most interesting part of the trip, was the river cruise.

6 . Its easy to see, even on the dreariest of days—how Paris has earned it”s reputation as the City of Love.

7 . Having decided to postpone her college education Jill began looking for a job.

8 . Regardless of who”s phone rings the entire class will be punished for any disruption.

9 . Isabella sprained her ankle, now she won”t be able to practice for several weeks.

10 . If you can”t take care of you”re own dog don”t expect me to pay for it”s grooming.

11 . Don”t expect this to be cheap, perfection has it”s price.

12 . What disappoints me most, is that you didn”t even tell me you were leaving.

13 . I told you, don”t go near the street!

14 . I remember that, The Monkey”s Paw , was my favorite short story in the ninth grade.

15 . The DVD”s that they just received, don”t seem to work in they”re player.

16 . A cyclotron, like the one Ernest Lawrence built at Berkeley—accelerates particles in a spiral path.

CHAPTER 4 ANSWER KEY

Exercise 1

1 . The team were was met .

2 . The flock look looks like a whirlwind .

3 . Carmen were was unaffected .

4 . Juggling seem seems too much .

5 . Others is are concerned .

6 . Every one has supported my decision . (correct)

7 . The fact have has forced some historians …

8 . The progression are a result of gradual modifications, not sudden overhaul . The subject and verb disagree, but more important, they are weak and unclear. Revision: We progress more by small increments than by major upheavals .

9 . The development were affected by the lack . The subject and verb disagree, but more important, they are weak and unclear. Revision: The discord within the revolutionary army hindered social and political development .

10 . This report is intended . Very uninformative subject and verb. Revision: The administration responded poorly to the most recent crises in the Middle East .

Exercise 2

1 . was

2 . were

3 . is

4 . goes

5 . wants

6 . was

7 . are

8 . is

9 . are

10 . are

11 . are

12 . have

13 . was

14 . have

15 . are

16 . go

17 . are

18 . was

19 . has

20 . was

21 . has

22 . utter

23 . are

Exercise 3

1 . The comedy The Return, the third and latest movie directed by H. K. Schaffer, daughter of famed screenwriter George Schaffer, has received widespread critical acclaim .

2 . An international team of scientists has discovered that the prefrontal cortex governs impulse control in humans, providing an important insight into criminal behavior .

3 . Although electric cars are widely considered to be environmentally friendly, the electricity they use often comes from power plants that burn coal or other fossil fuels, which generate copious greenhouse emissions .

4 . Regular exercise not only strengthens your muscles and heart, but also oxygenates your brain, helping it work more efficiently .

5 . Although we are motivated by our principles, those principles change as our experiences transform our priorities .

Exercise 4

1 . One of their greatest challenges, DNA contamination, has recently been overcome at the Max Planck Institute in Germany, where biologists have developed a “clean room,” like those used in manufacturing computer chips and space telescopes, to examine minute bits of genetic material from 400,000-year-old hominid bones .

2 . However, evidence suggests that the loss of autonomy and frequent humiliation that prisoners receive only aggravates the crime problem by exacerbating any short- or long-term psychological issues that make them susceptible to antisocial and criminal impulses .

3 . Nevertheless, the willingness to equate all governmental institutions with tyranny is an enormously dangerous one that can only impede moral, economic, and cultural progress .

Exercise 5

1 . … reforming the tax code, improving the schools, and reviving good relations

2 . … but about using your time well .

3 . … the more you will like her .

4 . … but also very reasonably priced .

5 . … exacerbated by esoteric financial instruments, skittish investors, and oblivious regulators .

6 . … is very supportive and knowledgeable .

7 . … give the tickets to Maria or to Caitlyn .

8 . … experienced contracting wealth, increasing risk spreads, and deteriorating credit markets .

9 . I prefer the romantic virtuosity of Liszt to the emotional accessibility of Chopin .

10 . … not so much for the music as for the spirit of free expression .

Exercise 6

1 . As I rounded the bend

2 . Martha was compelled by her creative instinct

3 . Claudius unsheathed his sword

4 . There are a lot of apps and websites to help you find a good Thai restaurant .

5 . Even though Adam had a sprained ankle, the coach forced him

6 . As we walked back to the car, we found my lost earrings .

7 . David was concerned primarily with

8 . the hikers pitched an early camp because of the looming storm

9 . Allison was frightened

10 . Without your being aware of it

11 . We were surprised to hear Carol, whom we always regarded as a dutiful mother ,

12 . Sprinters say that proper hip positioning is essential to getting a good jump out of the starting blocks .

13 . the town council is considering proposals for cutbacks

14 . the senator planned

15 . Although Portland is famous for its visual arts scene, its

16 . the magician made the coin disappear instantly

17 . Physicians recommend both vigorous exercise and disciplined eating for maintaining good health .

18 . we saw one at a garage sale

Exercise 7

1 . much strongermuch more strongly

2 . foullyfoul

3 . never usuallyrarely

4 . mostmore

5 . weren”t hardlyweren”t

6 . easiermore easily

7 . clearermore clearly

8 . more easyeasier

9 . the most uniqueunique

10 . won”t barelywon”t

11 . never tolddidn”t tell

12 . never usuallyrarely

Exercise 8

1 . anythinganything else

2 . keykeys

3 . American classroomsthose in American classrooms

4 . conventional carsthose of conventional cars

5 . as inscrutable, if not more so, thanas inscrutable as, if not more inscrutable than ,

6 . ShakespeareShakespeare”s poetry

7 . muchmany

8 . betweenamong

9 . a lifeguardlifeguards

10 . is you haveis having

Exercise 9

1 . wherewhen

2 . a student wantsyou want (or beginhe or she should begin )

3 . sheJulia (or Caroline )

4 . thatwho

5 . theirits, him or herthem

6 . itthey

7 . thatwho, theyhe or she

8 . oneyou (or youone )

9 . heJack (or Ted or whoever was missing the shots)

10 . his or hertheir

11 . wherein which

12 . EveryoneThey all (or theirhis or her )

13 . theyit

14 . thatwho

15 . each swimmerall swimmers (or themselveshimself or herself )

16 . thatwho

Exercise 10

1 . me

2 . he (did)

3 . We

4 . his or her

5 . us

6 . me

7 . him

8 . I

9 . our

10 . I

11 . him and me

12 . We

13 . me

14 . me

15 . his

16 . me

17 . your

Exercise 11

1 . came, had stayed

2 . having spent, has written

3 . Having developed, hoped

4 . went

5 . is, published

6 . Having found, had

7 . Having been, began

8 . arrived, started

9 . adjourned, had voted

10 . had not scored

11 . uses, based

12 . had, had hiked

13 . Having walked, were

14 . will have written

15 . Having won, was, would win

16 . is

17 . worry, forget

18 . have completed

19 . believed, were caused

20 . often worry

Exercise 12

1 . affect

2 . eradicate

3 . morale

4 . complements

5 . distinguish

6 . imply

7 . given

8 . affect

9 . tense

10 . principal

11 . disparaged

12 . compel

13 . antidote

14 . dilute

15 . exceeded

16 . converted

17 . inequity

18 . anonymously

19 . elicit

20 . adapt

21 . blossomed

22 . discreet

23 . council

24 . flout

25 . gamut

26 . infer

27 . curtail

28 . phase

29 . imminent

30 . proceed

31 . defusing

32 . reluctant

33 . supplanted

34 . cited

35 . redundancies: back, past, new, for the first time, entire, If and, necessarily, consistency and, on, at the same time, to accomplish the same thing, If we take a moment to pause and consider for a second

Exercise 13

1 . to

2 . on

3 . to

4 . none

5 . to (or for )

6 . from

7 . with

8 . about

9 . with

10 . into

11 . with

12 . about

13 . from

14 . of

15 . about

16 . to

17 . to

18 . none

Exercise 14

1 . were

2 . was, began

3 . would promote

4 . be

5 . were

6 . had, would have gotten

7 . were

8 . were

9 . had

10 . would

11 . were

12 . had, would have come

13 . be

14 . were

15 . had

16 . were

17 . wanted, see

18 . would have

19 . had, would have

Exercise 15

1 . … book , In Cold Blood, is considered the first and the greatest …

2 . delete the comma

3 . Runners who step out of their lanes during the first lap will be disqualified .

4 . … belief, water will reach its boiling point more slowly when it”s …

5 . … opinion, the most interesting part of the trip was …

6 . It”s easy to see, even on the dreariest of days, how Paris has earned its …

7 . … college education, Jill …

8 . … whose phone rings, the entire class …

9 . … ankle; now she …

10 . … your own dog, don”t expect me to pay for its grooming .

11 . Don”t expect this to be cheap; perfection has its price .

12 . delete the comma

13 . I told you: don”t …

14 . delete both commas

15 . The DVDs that they just received don”t seem to work in their player .

16 . cyclotron—like the one Ernest Lawrence built at Berkeley—accelerates …